Difference between revisions of "1951 AHSME Problems/Problem 37"

m (Solution)
Line 7: Line 7:
 
==Solution==
 
==Solution==
  
If we add <math>1</math> to the number, it becomes divisible by <math>10, 9, 8, \cdots, 2, 1</math>. The LCM of <math>1</math> throught <math>10</math> is <math>2520</math>, therefore the number we want to find is <math>2520-1=\boxed{\textbf{(D)}\ 2519}</math>
+
If we add <math>1</math> to the number, it becomes divisible by <math>10, 9, 8, \cdots, 2, 1</math>. The LCM of <math>1</math> through <math>10</math> is <math>2520</math>, therefore the number we want to find is <math>2520-1=\boxed{\textbf{(D)}\ 2519}</math>
  
 
== See Also ==
 
== See Also ==

Revision as of 13:11, 16 October 2016

Problem 37

A number which when divided by $10$ leaves a remainder of $9$, when divided by $9$ leaves a remainder of $8$, by $8$ leaves a remainder of $7$, etc., down to where, when divided by $2$, it leaves a remainder of $1$, is:

$\textbf{(A)}\ 59\qquad\textbf{(B)}\ 419\qquad\textbf{(C)}\ 1259\qquad\textbf{(D)}\ 2519\qquad\textbf{(E)}\ \text{none of these answers}$

Solution

If we add $1$ to the number, it becomes divisible by $10, 9, 8, \cdots, 2, 1$. The LCM of $1$ through $10$ is $2520$, therefore the number we want to find is $2520-1=\boxed{\textbf{(D)}\ 2519}$

See Also

1951 AHSC (ProblemsAnswer KeyResources)
Preceded by
Problem 36
Followed by
Problem 38
1 2 3 4 5 6 7 8 9 10 11 12 13 14 15 16 17 18 19 20 21 22 23 24 25 26 27 28 29 30 31 32 33 34 35 36 37 38 39 40 41 42 43 44 45 46 47 48 49 50
All AHSME Problems and Solutions

The problems on this page are copyrighted by the Mathematical Association of America's American Mathematics Competitions. AMC logo.png